K
Khách

Hãy nhập câu hỏi của bạn vào đây, nếu là tài khoản VIP, bạn sẽ được ưu tiên trả lời.

2 tháng 3 2020

\(A=\left(x+y+z+\frac{1}{4x}+\frac{1}{4y}+\frac{1}{4z}\right)+\frac{3}{4}\left(\frac{1}{x}+\frac{1}{y}+\frac{1}{z}\right)\)

\(\ge2\sqrt{x.\frac{1}{4x}}+2\sqrt{y.\frac{1}{4y}}+2\sqrt{z.\frac{1}{4z}}+\frac{3}{4}\left(\frac{9}{x+y+z}\right)\)

\(\ge1+1+1+\frac{3}{4}.\frac{9}{\frac{3}{2}}=\frac{15}{2}\)

Dấu "=" xảy ra <=> x = y = z = 1/2

Vậy min A = 15/2 tại x = y = z = 1/2

22 tháng 6 2020

Lời giải của em ạ :D

\(A=x+y+z+\frac{1}{x}+\frac{1}{y}+\frac{1}{z}\)

\(\ge x+y+z+\frac{9}{x+y+z}\)

Đặt \(t=x+y+z\le\frac{3}{2}\)

Khi đó \(A=t+\frac{9}{t}=\left(t+\frac{9}{4t}\right)+\frac{27}{4t}\ge3+\frac{27}{4\cdot\frac{3}{2}}=\frac{15}{2}\)

Đẳng thức xảy ra tại x=y=z=1/2

1 tháng 3 2018

b, Gọi biểu thức đề ra là B

=> Theo bđt cô si ta có : \(B\ge3\sqrt[3]{\left(x^2+\frac{1}{y^2}\right)\left(y^2+\frac{1}{z^2}\right)\left(z^2+\frac{1}{x^2}\right)}\)

=> \(B\ge3\sqrt[3]{2\cdot\frac{x}{y}\cdot2\cdot\frac{y}{z}\cdot2\cdot\frac{z}{x}}=3\sqrt[3]{8}=6\) 

( Chỗ này là thay \(x^2+\frac{1}{y^2}\ge2\sqrt{\frac{x^2}{y^2}}=2\cdot\frac{x}{y}\) và 2 cái kia tương tự vào )

=> Min B=6

1 tháng 3 2018

Theo bđt cô si thì ta có : \(\sqrt{\left(x+y\right)\cdot1}\le\frac{x+y+1}{2}\)

\(\sqrt{\left(z+x\right)\cdot1}\le\frac{z+x+1}{2}\)

\(\sqrt{\left(y+z\right)\cdot1}\le\frac{y+z+1}{2}\)

=> Cộng vế theo vế ta được : \(A\le\frac{2\left(x+y+z\right)+3}{2}=\frac{5}{2}\)

Dấu = xảy ra khi : x+y+z=1 và x+y=1 và y+z=1 và x+z=1

=> \(x=y=z=\frac{1}{3}\)

Vậy ...

17 tháng 5 2017

Bất đẳng thứ côsi hả bạn

17 tháng 5 2017

Mình sửa lại đề nhé:

\(\frac{x}{x^2+1}+\frac{y}{y^2+1}+\frac{z}{z^2+1}\le\frac{3}{2}\le\frac{1}{1+x}+\frac{1}{1+y}+\frac{1}{1+z}\)

Dễ dàng chứng minh được: \(x^2+1\ge2x\Leftrightarrow\frac{x}{x^2+1}\le\frac{x}{2x}=\frac{1}{2}\)

Tương tự, ta cũng có: \(\frac{y}{y^2+1}\le\frac{1}{2};\frac{z}{z^2+1}\le\frac{1}{2}\)

Cộng từng vế của 3 BĐT trên ta được ĐPCM.

Ta chứng minh BĐT: \(\frac{1}{a}+\frac{1}{b}+\frac{1}{c}\ge\frac{9}{a+b+c}\Leftrightarrow\left(a+b+c\right)\left(\frac{1}{a}+\frac{1}{b}+\frac{1}{c}\right)\ge9\)

\(\Leftrightarrow3+\left(\frac{a}{b}+\frac{b}{a}\right)+\left(\frac{b}{c}+\frac{c}{b}\right)+\left(\frac{a}{c}+\frac{c}{a}\right)\ge9\)

\(\Leftrightarrow\left(\frac{a}{b}+\frac{b}{a}\right)+\left(\frac{b}{c}+\frac{c}{b}\right)+\left(\frac{c}{a}+\frac{a}{c}\right)\ge6\)

BĐT này đúng với \(\frac{a}{b}+\frac{b}{a}\ge2\)

Áp dụng BĐT \(\frac{1}{a}+\frac{1}{b}+\frac{1}{c}\ge\frac{9}{a+b+c}\), ta được:

\(\frac{1}{1+x}+\frac{1}{1+y}+\frac{1}{1+z}\ge\frac{9}{3+x+y+z}\ge\frac{9}{3+3}\ge\frac{3}{2}\)

1 tháng 7 2017

\(A=\frac{1}{\sqrt{x}}+\frac{1}{\sqrt{y}}+\frac{1}{\sqrt{z}}\ge\frac{2}{x+1}+\frac{2}{y+1}+\frac{2}{z+1}\ge\frac{18}{x+y+z+3}=3\)

2 tháng 7 2017

cảm ơn nha

9 tháng 6 2018

Sử dụng BĐT AM-GM, ta có: 

\(x^3+y^2\ge2yx\sqrt{x}\)

\(\Rightarrow\frac{2\sqrt{x}}{x^3+y^2}\le\frac{2\sqrt{x}}{2yx\sqrt{x}}=\frac{1}{xy}\)

Tương tự cộng lại suy ra: 

\(VT\le\frac{1}{xy}+\frac{1}{yz}+\frac{1}{zx}\le\frac{1}{x^2}+\frac{1}{y^2}+\frac{1}{z^2}\)

BĐT Bunhiacopxky em chưa học cô ạ

Cô cong cách nào không ạ

AH
Akai Haruma
Giáo viên
1 tháng 6 2020

Nguyễn Thị Nguyệt Ánh:

Vậy thì bạn có thể chứng minh $\frac{1}{x}+\frac{1}{y}+\frac{1}{z}\geq \frac{9}{x+y+z}$ thông qua BĐT Cô-si:

Áp dụng BĐT Cô-si:

$x+y+z\geq 3\sqrt[3]{xyz}$

$xy+yz+xz\geq 3\sqrt[3]{x^2y^2z^2}$

Nhân theo vế:

$(x+y+z)(xy+yz+xz)\geq 9xyz$

$\Rightarrow \frac{xy+yz+xz}{xyz}\geq \frac{9}{x+y+z}$
hay $\frac{1}{x}+\frac{1}{y}+\frac{1}{z}\geq \frac{9}{x+y+z}$

19 tháng 7 2017

Áp dụng BĐT AM-GM ta có:

\(\frac{\sqrt{1+x^3+y^3}}{xy}\ge\frac{\sqrt{3\sqrt[3]{x^3y^3}}}{xy}=\frac{\sqrt{3xy}}{xy}=\frac{\sqrt{3}}{\sqrt{xy}}\)

Tương tự cho 2 BĐT còn lại ta có:

\(\frac{\sqrt{1+y^3+z^3}}{yz}\ge\frac{\sqrt{3}}{\sqrt{yz}};\frac{\sqrt{1+z^3+x^3}}{xz}\ge\frac{\sqrt{3}}{\sqrt{xz}}\)

Cộng theo vế 3 BĐT trên ta có:

\(M\ge\sqrt{3}\left(\frac{1}{\sqrt{xy}}+\frac{1}{\sqrt{yz}}+\frac{1}{\sqrt{xz}}\right)=\sqrt{3}\cdot\left(\frac{\sqrt{x}}{\sqrt{xyz}}+\frac{\sqrt{y}}{\sqrt{xyz}}+\frac{\sqrt{z}}{\sqrt{xyz}}\right)\)

\(=\sqrt{3}\cdot\frac{\sqrt{x}+\sqrt{y}+\sqrt{z}}{\sqrt{xyz}}\ge\sqrt{3}\cdot\frac{3\sqrt[3]{\sqrt{xyz}}}{1}=3\sqrt{3}\)

Khi \(x=y=z=1\)